triplet pythagoricien. - Page 2
Répondre à la discussion
Page 2 sur 2 PremièrePremière 2
Affichage des résultats 31 à 57 sur 57

triplet pythagoricien.



  1. #31
    invitea8961440

    Re : triplet pythagoricien.


    ------

    De si jeunes talents dans la démonstration de la conjecture qui a tenu en échec pendant 350 ans la société des mathématiciens de ce monde,et qui a meme pris la vie à certains ,le mathématicien japonais(...),j'aimerais bien voir la démonstration ,seulement il y a des préalables pour prétendre pouvoir démontrer une
    conjecture!

    -----

  2. #32
    invitec7b3f097

    Re : triplet pythagoricien.

    Citation Envoyé par ulrich richarovitch
    De si jeunes talents dans la démonstration de la conjecture!
    Ben Wiles est plutot vieux pour une telle reussite: il est connu que les meilleures années des mathématiciens sont leur premières.

    " qui a meme pris la vie à certains ,le mathématicien japonais"
    Euh lequel ?
    Tanyiama ou Shimura ?

    "j'aimerais bien voir la démonstration"
    cf http://math.stanford.edu/~lekheng/flt/wiles.pdf

    Sinon je fais mon TPE sur le Grand Théorème de Fermat, vous pouvez regarder le powerpoint:
    http://shadowlord.free.fr/TPE/fermat/

    On y trouvera une démonstration élémentaire du cas n=4.

  3. #33
    invitea8961440

    Re : triplet pythagoricien.

    Il s'agit de la démonstration que Leg aimerais donner et non de celle que l'on connait.

  4. #34
    leg

    Re : triplet pythagoricien.

    Citation Envoyé par ulrich richarovitch
    Il s'agit de la démonstration que Leg aimerais donner et non de celle que l'on connait.
    bonjours je pensais que tu avais lu tous les postes , le cas N = 4 est démontré de quatre manières différentes :
    si z est un carré alors x ne peut en être un!
    si z est un carré alors y ne peut en être un!
    en plus de l'absence du cas x et y ne peuvent être deux carrés et l'absence de surface carré d'un triangle rectangle qui abouti aussi à labsence de solution dans N = 4, mais peut être que tu pourrais démontrer si z est un carré alors x ou y n'en ai pas un , sans bien sur utiliser mes 2 démos quand bien même la forme ne te conviendrais pas..! donc pas de solution tel que z^4 = y² + x^4 ou z^4 = y^4 +x² d'où pas de solution dans N = 4 , puisque dans un Triplet P, il ne peut y avoir qu'un et un seul carré!

  5. #35
    leg

    Re : triplet pythagoricien.

    Sinon je fais mon TPE sur le Grand Théorème de Fermat, vous pouvez regarder le powerpoint:
    http://shadowlord.free.fr/TPE/fermat/
    bonjour ; (sur ton site concernat le tpe on a sans arrét des message d'avertissement virus ) ce n'est pas grave .
    dans la démos du cas N = 4 tu arrives a cette partie :
    il existe deux entiers tel que x² = u² -v² et z² = u² + v² il s'agit donc de deux solutions pythagoriques par addition et soustraction ce qui est absurde sans que u et v change de valeur un T.P = (triplet Pythagoricien) , n'a qu'une solution soit:
    x² + y² = z² qui est bien vrai = u² + v².
    mais x² - y² = ?d'où u² - v² = un entier non carré la démos est terminée x^4 + y² = z^4 n'existe pas ou alors on admet que la formule qui donne l'infinité des T.P peut donner deux T.P sans que P et q change de valeur ce qui est tout aussi absurde car en effetlorsque la solution existe u² + v²= z² alors par soustraction il s'agit de z² - v² = u² or il est évident que Z n'est pas égale à U ..! d' où deux carré ne peuvent donner deux carrés par addition et soustraction ! Fermat a dit si on donne deux carrés alors on donne deux autres carrés ayant la même propriété ...etc ..etc
    il n' a jamais précisé de quel carré il s'agissait car la descente infinie se produit dans les trois cas si z² et x² existe , si z² et y² existe ou si x² et y² existe il existerait indéfiniment deux autres carrés < indéfiniment en entier ce qui est impossible
    Fermat n'a jamais publié sa démos du cas N = 4 mais son raisonnement .
    je pense qu'il a utilisé la propriété de la fromule des T.P pour résoudre les 4 manières du cas N = 4
    dont celle ci qui est contradictoire u² - v² = x² soit (p² + q²) = u; 2pq = v et bien sur p² - q²= x avec p et q ayant bien les propriétés de ton tpe. premier entre eux et de p

  6. #36
    leg

    Re : triplet pythagoricien.

    suite arité différente.
    d'où la contradiction évidente
    (p² + q²) + (2pq) ne peut redonner un carré!p²+q² n'est pas égale p² - q² par conséquent u et v sont obligé de changer de valeur en fonction de l'addition ou de la soustraction. il n'était pas obligatoire de continuer ou d'utilisér le raisonnement de descente infinie ce qui me surprend pour qu'elle raison on ne c'est pas aperçu de cette contradiction.
    un mathematicien prof.. , de l'université de nice m'a déja répondu. effectivement il s'agissaitd'une érreur qui est passé ianparçu du fait que l'on utilisait une méthode pour abutir au même résultat mais en utilisant le raisonnement de Fermat .
    combien de mathématiciens ont utilisé la propriété de la formule qui donne les T.P ?
    a part Fermat qui je suppose l'a utilisé. pour résoudre le cas générale des puissances paires.
    et je ne pense pas le fait que je soit amateur ou néophite en la matière et que tout les mathématiciens depuis 350 ans n'on pas trouver la soltuion a part A Willes , que c'est suffisant pour apporter une critique sur l'impossibilité de solution par Férmat ou un autre , car pour cela il faudrait avoir déja démontré le cas N = 4 des quatres manières différentes,ce qui n'a jamais été publié et fait par qui que ce soit, notamment le cas que je vient d'expliquer! alors je vous laisse le soin, a votre tour de démontrer que mes 4 démos sont faussent !de maniére élémentaire bien sur. puis l'apporterais la contradiction du cas N = 3 et qui s'étend de façon générale à l'ensemmble des puissance premères! il devrait être facile d'analyser mes 4 postulats même si cela n'est pas écrit de façon conventionnellle mais c'est largement compréhensible pour les Matheux que vous êtes..! d'autres l'on déja analyser et n'ont pas contredit mais cela ne veut rient dire..c'est bien le but de ce forum faitre éclater la vérité et rendre a Fermat ce qui lui appartient sa solution ou du moins la possibilité qu'il l'avait en utilisant les outils mathématiques de son époque..A vous et merci pour votre aide.

  7. #37
    leg

    Re : triplet pythagoricien.

    Citation Envoyé par ulrich richarovitch
    j'aimerais bien te voir à la tache Leg pour tes démonstrations de cas n=4,n=6 ou encore n impair,amusant.
    bonsoir u-richarovitch; Y.Tanyama n'est pas mort a cause du théorème de Fermat mais de l'impossibilité de démontrer leur conjecture : Tanyama-Shimura; ce que A.Wiles a fait, et donc cela démontre de façon générale le T de Fermat.
    si tu connais la démos du cas N = 4 que tu as appris, et qui est publiée depuis des lustres, peut être que tu pourrais nous dire d'une autre façon si Z est un carré pourquoi Y ne peut être un carré dans un T.P tel que : Z' = p² + q²; et Y' = 2pq ce qui te permetrais de démontrer le cas N = 4 n'a pas de solution! différement de la démonstration si X est un carré alors Y n'en est pas un, ni un demi carré; ce qui changera un peu depuis trois siecles
    tu as deux possibilités;
    a)
    soit tu finis par trouver la contradiction par un raisonnement de descente infinie il existerait une autre solution plus petite avec toujours Z et y carrés < Z' etY' pour cela , analyse bien la propriété de p et q qui sont premier entre-eux et de parité différente...cette contradiction existe!
    b)
    soit tu trouves la contradiction, toute aussi élémentaire, mais tellement évidente en utilisant une propriété du théorème de Pythagore; que tout élève de sixième connait..amusant non..?
    là je vais encore t'aider: p² + q² - 2pq = ? autrement dit, p² + q² = ... + ... ; et pense au carré de l'hypoténuse...
    à moins bien sur que tu me démontre que le théorème de Pythagore est faux.!dans ce cas ma conjecture serait fausse, bien entendu; mais malheureusement il me restera la contradiction: a)

    donc dans un T.P si Z est un carré alors Y ne peut en être un ni même un demi carré; d'où z^4 = y^4 + x² n'existe pas et par là même z^4 = y^4 + x^4 non plus !

  8. #38
    invitea8961440

    Re : triplet pythagoricien.

    Moi ,je trouve toutes ces démonstrations trop longues,soit on admet que la preuve de Wiles est la seule du cas général ,soit on résoud de façon élementaire et concise le cas général. Ce qui importe en mathématiques,c'est la concision et bien sur la véracité des arguments !

  9. #39
    invitea8961440

    Re : triplet pythagoricien.

    Est-ce que 350 ans est réductible à "quelques années"(...) .

  10. #40
    leg

    Re : triplet pythagoricien.

    Citation Envoyé par ulrich richarovitch
    Moi ,je trouve toutes ces démonstrations trop longues,soit on admet que la preuve de Wiles est la seule du cas général ,soit on résoud de façon élementaire et concise le cas général. Ce qui importe en mathématiques,c'est la concision et bien sur la véracité des arguments !
    je pense que tu te contredis, car la démonstration de A. Wiles fait 200 pages, et n'est comprise que par une douzaine de mathématiciens dans le monde, ce qui est loing d'être générale; mais, sincéres félicitations si tu es parmis cela..ceci dit ce n'est pas le sujet du fil, ma question reste toujours valable pour toi, ou pour un autre la contradiction b) ne nécéssite que trois lignes, alors un petit éffort et svp, ce n'est pas tres élégant de noyer le poisson de cette façon. alors a + et merci pour ton aide et ta contribution si tu le veux bien. leg

  11. #41
    martini_bird

    Re : triplet pythagoricien.

    Bonsoir,

    je me permets quelques commentaires, et je vous prie de me pardonnez si ceux-ci vous paraissent hors-sujet.

    En premier lieu, je dirai qu'il y a un réel intérêt à démontrer le théorème de Fermat-Wiles de manière élémentaire (tout comme Selberg et Erdös l'ont fait pour le théorème des nombres premiers).

    Ceci étant, si une démonstration antérieure à celle de Wiles avait été valide, cela se serait su! (rappelons que Wiles n'est "que" le dernier maillon d'une longue chaîne).

    Je pense sincèrement que c'est un mythe de considérer que seul une poignée de mathématiciens est capable de comprendre et d'apprécier la démonstration de Wiles: la théorie des nombres et la conjecture de Shimura-Taniyama-Weil font (faisaient?) partie des thèmes privilégiés de la recherche en mathématique.

    Toutefois, j'avoue en toute franchise me perdre dans les explications obscures et sybillines de leg. Leg, si tu accordais un peu plus de temps à rédiger un discours cohérent et lisible, on te comprendrais mieux et tu gagnerais beaucoup en crédibilité! Je crois que les remarques d'Ulrich sont nées de ce fait.

    En conclusion, puisque nombre de mathématicien se sont penchés sur la question, rien ne nous empêche de rêver... Avec un minimum de retenue néanmoins.

  12. #42
    invitec7b3f097

    Re : triplet pythagoricien.

    "bonsoir u-richarovitch; Y.Tanyama n'est pas mort a cause du théorème de Fermat mais de l'impossibilité de démontrer leur conjecture : Tanyama-Shimura; ce que A.Wiles a fait, et donc cela démontre de façon générale le T de Fermat."

    Faux !!!

    Taniyama s'est suicidé et tout le monde en ignore la cause (même Shimura)

    Wiles n'a prouvé pas la conjecture STW dans le cas général mais dans le cas des courbes elliptiques rationnelles je crois.

  13. #43
    leg

    Re : triplet pythagoricien.

    bonsoir martini et lord,il est vrai que mes explications sont obscures;
    mais pour le cas N = 4 c'est largement compréhensible!
    si on s'intérresse uniquement au cas : si z² existe alors y² ne peut exister d'où (z²)² - (y²)² = x² n'existe pas donc pas de solution possible tel que (z²)² = (y²)² + (x²)²
    dans un T.P , donné par p et q de parité différente et premier entre-eux; z = p² + q² ; y = 2pq et x = p² - q²

    le théorème de pythagore nous dit le carré de l'hypoténuse est = à la somme des carrés des côtés de l'angle droit. Soit:
    (p² + q²)² = (2pq)² + (p² - q²)² .

    Mais ( p² + q²) - (2pq) = (p - q)² autrement dit Z - Y = d²pour tout Triplet P.. primitif

    par conséquent dans un T.P, primitif, si Z est un carré est Y aussi, nous somme bien confronté à une contradiction ..!

    le théorème de pythagore à une nouvelle solution!

    le carré de l'hypoténuse est = au carré d'un côté de l'angle droit : Y, et au carré de sa différence avec Z ce qui est absurde;

    soit : (p² + q²)² = (2pq)² + (p - q) ² ..? au lieu de (p² - q²)²

    le plus petit T.P ne serait plus 3, 4 et 5 mais la solution 1 + 4 = 5 , 1 +12 = 13 ...etc etc

    d'où dans un T.P primitif si Z est un carré alors Y n'en est pas un, alors il ne peut exister de solution dans N = 4 !

    c'était une partie de la question à résoudre , car il y a une deuxième contradiction dans cette supposition si Z et Y sont carré, en éffet:
    tout d'abord, si p² + q² = Z carré, c'est bien parce que p et q sont pythagorique P = x et q = y il existe donc p' < p et q' < q qui vont nous donner par obligation p = x et q = y afin que x² + y² = Z²

    or si Y est aussi un carré = à 2pq = 2xy cela correspond bien à légalité suivante :
    [ 2 (p'² - q'²) (2 p' q') ] = Y²
    p' et q' ont la même propriété que p et q mais plus petits.
    2 * 2 = 4
    p' * q' = a² si le produit de deux entiers premiers entre-eux est un carré alors ce sont deux carrés...etc
    p'² - q'² = x² car a est pair et x impair même chose...etc

    donc x² * a² * 4 = Y²! Mais alors si p'² - q'² = x²; p' et q' sont encore pythagorique < à p et q mais aussi < à Z et Y ; c'est à dire :
    z' < Z ; y' < Y et comme p' et q' sont carrés;
    on se retrouve à la solution de départ, avec deux autres carrés inferieur ayant la même propriété...etc etc .
    donc A+ pour vos commentaires et vos remarques merci . leg

  14. #44
    martini_bird

    Re : triplet pythagoricien.

    Citation Envoyé par leg
    le plus petit T.P ne serait plus 3, 4 et 5 mais la solution 1 + 4 = 5 , 1 +12 = 13 ...etc etc
    Bonsoir leg,

    5, 12 et 13 sont des carrés? :confused:

    Il y a quelque chose que j'ai pas suivi...

  15. #45
    leg

    Re : triplet pythagoricien.

    voyons martini, 1 + 12 =13 tu pousses un peu...si (p - q) était solution au lieu de
    p² -q² pour p = 4 et q = 3 au lieu du T.p; p² - q² = x = 5 ; 2pq = Y = 12 et
    p² + q² = Z =13
    on aurait la solution p - q = d =1 puis y = 12 et z = 13 soit 1+12=13 au lieu
    de 25 +144=169
    un T.P n'est jamais une solution sans être mis au carré! c'est comme si tu me disais que l'hypoténuse est = à la somme des côtés de l'angle droit..

    tu connais la clef du théorème de Pyt .. un grand carré de côté C et un carré< à l'interieur,de côté Z, qui donne 4 triangles rectangles d'hypoténuse Z
    C²= [(a^n)² + (b^n)²]+ 2 (a^n)(b^n) = (a^n + b^n)²

    la racine carreé = C d'où pour n >1,
    (a^n)² + (b^n)² ne peut jamais être un carré ni un carré au cube , d'une maniére générale, ni un carré élevé à la puissance premiére!sinon on aurait une solution dans
    N = 4, 6 ,10...

    si n et > ou = à 2 regarde avec n = 2, 3 , 5 , 7 ainsi que a = 2 ; b =1 et rajoute 1 à a et b analyse les valeurs
    A+

  16. #46
    martini_bird

    Re : triplet pythagoricien.

    Salut,

    ok leg, je vois ce que tu veux dire.

    Citation Envoyé par martini_bird
    Il s'agit donc de démontrer que l'équation n'a pas de solution entière non triviale.
    Une fois que ceci est fait rigoureusement, on a le théorème de Fermat pour tous les entiers pairs.

    Maintenant, je vois pas comment faire pour adapter la situation à d'autres entiers.

    Cordialement.

  17. #47
    leg

    Re : triplet pythagoricien.

    bonsoir martini; pour " adapter la situation à d'autres entiers") tu veux bien dire des entiers impairs et premiers .

    Fermat étant plus rigoureux que moi, il avait donc la solution générale des entiers pairs. Car il avait largement la possibilité de démontrer les trois cas :

    dans un Triplet Pythagoricien, si X est un carré , alors Z n'en est pas un ni Y, de plus Y ne peut non plus être un demi carré

    si Z est un carré , alors Y n'en est pas un ! ni un demi carré.

    de plus il ne peut exister un T.P , primitif donné par p et q non entier
    donc pas de solution dans N = 2, tel que X^4 + Y^4 = Z² ;ou X^4 + y² = Z^4
    ou encore X² + Y^4 = Z^4. sinon la démo des puissance paires serait fausse ainsi que le raisonnement de descente infinie; il pourrait exister une des trois solutions impossible! cette particularité est importante car elle indique l'absence de solution dans la première puissance est première : N = 2.!

    Or pour avoir un T.P dans les racines carrées d'une puissance première > 2 par ex ,3 ,5 ,7, 11...etc il nous faudrait impérativement p et q non entier pour avoir au moin une chance d'en trouver un
    par exe: P = 2,86151111...etc et q = 1, 7297670...ect donne le T.P
    racine carrée de X^3; Y et Z^3 soit la solution mis au carré: 3^3 + 98 = 5^3 .

    On va donc supposer qu'il existe une solution dans une de ces puissance, donc un T.P dans leur racine carrée, comme pour N = 2 ce qui est logique! on ira donc choisir p et q dans ces racines, toujours comme pour N = 2 , afin d'avoir au minimum , cette solution qui existe par supposition ; soit par addition oupar soustraction car comme pour les puissances paires :
    deux carrés ne peuvent donner un carré par addition et soustraction ;(démo N°2)
    deux cubes ne peuvent donner un cube par addition et soustraction (démo N° 2)
    plus généralement deux produits de puissance N , ne peuvent donner un produit de puissance N par addition et soustraction sans que p ou q ne change de valeur !

    On verra les conséquences

    Euler à démontré le cas N = 3 celà indique que quelque soit la nature de p et q ,
    ils ne donne pas de T.P dans les racines carréesc'est une autre démo et un autre raisonnement mais qui pourra nous servir . je pense que Fermat avait aussi résolu le cas N=3 pour confirmer sa Démonstration générale des puissances premières
    A + et amicalement Martini ,
    ( je te laisse réfléchir avant la suite et afin de bien comprendre le role et la propriété de cette formule qui donne les T.P .)

  18. #48
    martini_bird

    Re : triplet pythagoricien.

    Bonsoir,

    je vais paraître têtu, mais bon...

    Les triplets pythagoriciens donnent les solutions du cas N=2. On peut effectivement s'en servir pour démontrer que si N est pair, le problème de Fermat n'admet pas de solutions non triviales. En effet, dans ce cas l'équation peut s'écrire et les nombres Xn, Yn et Zn forment un TP.

    Maintenant, si on s'intéresse au cas N=3 (dans un premier temps), je ne vois pas sur quelle base on peut utiliser les TP? :confused:

  19. #49
    leg

    Re : triplet pythagoricien.

    Citation Envoyé par martini_bird
    Bonsoir,

    je vais paraître têtu, mais bon...

    Les triplets pythagoriciens donnent les solutions du cas N=2. On peut effectivement s'en servir pour démontrer que si N est pair, le problème de Fermat n'admet pas de solutions non triviales. En effet, dans ce cas l'équation peut s'écrire et les nombres Xn, Yn et Zn forment un TP.

    Maintenant, si on s'intéresse au cas N=3 (dans un premier temps), je ne vois pas sur quelle base on peut utiliser les TP? :confused:
    bonsoir martini ,non tu n'est pas têtu et le contraire serait tout à fait justifié. je vais utiliser les TP en les choisissant dans les racine carreés des cubes comme pour N = 2 il y a des solutions dans cette puissance donc des TP dans leur racine carrée afin d'utiliser la démo N°2 et faire ressortir qu'il ne peut y avoir deux cubes au carré par addition et soustraction...faire ressortir la même contradiction que N=6 je fait un copi coller: sauf pour K0 , K1 et K2 qui veut dire N°0,1 et 2
    tous les autres a6 a3,b3 y3 ..etc veut dire a^6 ou 3 idem pour b, c ,x et z

    Mais alors si : il existe a^6 + b^6 = c6 , il existe :
    (p² -q²)² = X^3, avec X carré, soit : √X^3 = a^3 et évidement √X = a et a^6 = X^3
    (2 p q)² = Y^3, avec Y carré, soit : √Y^3 = b^3 et évidement √Y = b et b^6 = Y^3
    (p² + q²)² = Z^3, avec Z carré, soit : √Z^3 = c^3 et évidement √Z = c et c^6 = Z^3 ; il est clair qu’il s’agirait d’une solution multiple d’un T.P primitif de trois carrés !, tel que :
    (p² -q²)² = X^3, = K0 (p² - q²) = X^3 et avec K0 = x ; (2 p q)² = Y^3, = K1 (2 p q) = Y^3, avec K1 = y ; et enfin, (p² + q²)² = Z^3, = K2 (p² + q²) = Z^3, avec K2 = z , "à première vue…"
    « et où bien évidement les facteurs K, sont trois entier carrés élevés à la puissance n » impliquant la solution : X3 + Y 3 = Z3. soit : K0 * a^4 + K1 *b^4 = K2 *c^4

    C'est-à-dire trois facteurs distincts et ce quelque soi la puissance N>2 choisi, en commençant par N = 4, 6 , 10 , 14 …etc. On ne pourrait traiter que les puissances paires.
    (A l’évidence si A implique B , non B implique non A !)
    Or on sait que la formule donne des T.P multiple d’un primitif, ayant le facteur
    K >1, commun pour X, Y et Z et non un facteur distinct ; mais supposons qu’il existerait dans une puissance paire > 2, cette solution avec un facteur distinct ; alors qu’elle est la nature du T.P primitif X, Y et Z … ? On vient de le voir :
    Trois carrés, ce qui est absurde Fermat en a démontré l’impossibilité, et on sait depuis plus de 3 siècles et demi, que dans un T.P primitif X , Y et Z il ne peut y avoir un et seulement un seul carré … ! Alors de toute évidence il ne s’agirait pas d’un T.P primitif : X, Y et Z d’où la notation est exclu tel que :
    X² = a4 mais ≠ p² - q² sinon cela voudrait dire que X² = a3 !
    Y² = b4 ≠ 2pq idem Y² ≠ b3 et pareil pour Z² = c4 ≠ p² + q² ; Z² ≠ c3 !
    Mais : venons en au T.P :X = a3 , Y = b3 et Z = c3
    X² est un cube au carré = (z + y) (z – y), propriété pythagorique ces deux nombres sont des carrés et aussi des cubes sans pour autant que z et y soient carrés, or on a démontré que si le produit de deux entiers , de parité différente est premiers entre-eux deux à deux est un cube ces deux nombres sont aussi des cubes et z et y sont des cubes par supposition !
    De plus z – y = (p – q)² soit un cube au carré = (v3)²… ?

    Z3 + y3 = (u3)² et z3 – y3 =(v3)² et [(u3)² (v3)²] = X² = (a3)²
    Soit : (p’² +q’²)² + (2p’q’)² = (p’² + q’²)².. ? contradiction et impossibilité, reste
    (p’² +q’²)² - (2p’q’)² = (p’² - q’²)² = z3 - y3 = (v²)3
    Donc deux autres solutions cubiques , inférieures et plus petites indéfiniment, comme le cas N = 4 , démo n°2 ; par addition et soustraction : impossible !

    il en serait pareil avec N = 5, 7 , 11 , 13…etc
    comment peut on supposer encore, qu’il pourrait y avoir un solution dans une puissance paire > 2 .. ? Il n’y a donc pas de solution d’une manière générale générale dans une puissance paire >2 !

    bonsoir martini je vais

  20. #50
    leg

    Re : triplet pythagoricien.

    encore un dernier mot : suite cela veut dire qu'il n'exsite aucun TP dans les puissances N premiéres mais aussi dans les racines carrées entières , de ces entiers élevés à la puissance N > 2 et premiéres ! A+

  21. #51
    leg

    Re : triplet pythagoricien.

    Bonjour, à tous et slu Martini-B. Voilà un Rappel et une partie de la suite.

    RAPPEL :

    a) Si une équation donne un carré par addition ou soustraction, ces deux nombres sont Pythagoriques ! Tel que X² + Y² = Z² soit :
    (p² - q²)² + (2pq)² = (p² + q²)²

    b) on ne peut noter ou dire, que le carré de l’hypoténuse est égal à la somme de deux carrés ; dont l’un, est le côté du triangle rectangle en l’occurrence Y et l’autre est sa différence avec l’hypoténuse Z, soit : d² = (p² + q²) – (2pq). Le théorème de Pythagore ne voudrait plus rient dire.
    Démo n° 1 si Z est un carré, Y n’en est pas un !

    c) On ne peut noter, que deux carrés donnent un carré par addition et soustraction, ou 2 cubes, et plus généralement deux produits de puissance N >1.

    Si X² + Y² = Z² ceci est égal à : (p² – q²)² + (2pq)² = (p² + q²)² donc on ne peut dire X² - Y² = u² soit :
    (p² – q²)² - (2pq)² = (p’² - q’²)²..? cela voudrait dire que :
    (p² – q²)² = (p² + q²)², d’où X doit changer de valeur et être égal à :Z = p² + q²
    Démo n° 2 si X est un carré, Z n’en est pas un !

    d) Si deux cubes au carré, donnent un cube par addition ou soustraction, ces deux cubes sont pythagoriques, pareil pour deux produits de puissance >3.
    On peut noter que: (X²)^3 + (Y²)^3 = (Z²)^3 est égal à :
    (p² - q²)² + (2pq)² = (p² + q²)², Mais cela ne veux pas dire que X² = p² – q², sinon : X² = X^3
    or : p² - q² = X^3 mais on note l’égalité suivante, (X^3)² + (Y^3)² = (Z^3)², qui à le même résultat ; pareil pour toutes les puissances premières mises au carré.
    Le triplet x², y² et z² n’est donc pas pythagorique ; mais cela serait un bon moyen, pour contourner le problème si cette égalité avait une solution, c'est-à-dire si deux carrés étaient somme d’un carré, sans qu’ils soient pythagoriques, mais en utilisant trois facteurs carrés x ,y et z , tel que :
    (x x²) + (y y²) = (z z²) qui à l’égalité suivante, x^6 + y^6 = z^6 pythagorique!

    e) Ce sont ces différentes propriétés, pour démontrer l’absence de solution générale dans les puissances paires que j’utilise. Une autre propriété des T.P, le produit de (Z + Y) (Z – Y) est égal à X carré, quand bien même Z et Y ne serait pas deux carrés, puisqu’ils sont Pythagoriques !


    Conclusion générale :
    Probablement le raisonnement de P de Fermat.

    Lorsque l’on choisit les deux nombres p et q dans les entiers naturels, racine carrée de ces entiers à la puissance deux, pour obtenir un T.P dans ces mêmes entiers, et une solution x² + y² = z² il n’y a aucune interdiction !
    Cela ne sera plus du tout le cas dans les racines carrées des puissances impaires et première.
    En effet cela donnerait une solution par addition et soustraction quel que soit p et q choisis dans les racines carrées entières. Ce qui est impossible et démontré.
    Alors que ce n’est pas du tout le cas pour N = 2 ; je peux choisir p et q étant deux carrées quelconques, ou pythagoriques, pour obtenir n’importe quel T.P, et en mettant p et q au carré, je ne pourrais avoir au maximum : qu’une solution soit par addition soit par soustraction ; tel que ceci a été démontré ! Si je choisis p et q entre les entiers naturels alors je n’ai aucune solution dans N = 2.

    Pour trouver l’infinité des T.P ayant une solution dans N = 2, T.P qui par obligation se trouvent dans N = 1 et uniquement dans N = 1. Nous avons du choisir p et q dans les racines carrées des entiers, élevés à la puissance N = 2 ; c'est-à-dire dans N = 1 ! p = 2 et q = 1 les deux plus petits.

    Il devient évident puisque les solutions existent dans N = 2, de trouver X² ou Z² soit par addition, soit par soustraction, en choisissant p et q, dans les racines carrées des entiers élevés a cette puissance 2. Autrement dit, à un moment donné p et q sont choisis dans un T.P qui existe ! Par ex : p = 3, et q = 4 qui mis au carré, me donnent par addition la solution d’un T.P qui existe. Inversement : p = 5 et q = 4, me donne la même solution du T.P, mais par soustraction, « p change de valeur ».
    Nous savons que p et q non entiers, ne donnent pas de T.P primitif ayant une solution dans N = 2.
    Alors si une solution existe, dans une puissance N ≥ 3, en choisissant p et q dans les racines carrées d’une de ces puissances, on aura obligatoirement les solutions, par addition ou par soustraction tel que : p² - q² = x^N ou p² + q² = z^N ; soit : z^N – y^N = x^N, puisque p = √z^N et q = √y^N ; inversement : x^N + y^N = z^N, car p = √x^N et q = √y^N. Exactement comme dans N = 2. (ainsi que la démo n°2 du cas N = 4.)Donc quelque soit la puissance première > 2 choisis, si une autre solution existait elle serait de la forme : (x^N)² + y^N = (z^N)² ou (z^N)² - y^N = (x^N)², au minimum !
    « En plus de la solution où p et q ont été choisis. IL suffirait ensuite de multiplier par un facteur commun, soit le T.P ou la solution pour en avoir une infinité.»

    P de Fermat c’est arrêté au cas N = 3, prenons cet exemple, il est général.
    Si une solution existe dans N = 3 , p et q étant choisis dans les racines carrées, de ces entiers à la puissance 3, nous allons tomber sur la solution qui existe, soit par addition soit par soustraction. p = √z^N et q = √y^N .

    A + + je vous laisse pour ce jour a vos réflections. leg

  22. #52
    leg

    Re : triplet pythagoricien.

    bonsoir à tous.
    suite poste 51.

    P de Fermat c’est arrêté au cas N = 3, prenons cet exemple, il est général. Si une solution existe dans N = 3 , p et q étant choisis dans les racines carrées de ces entiers à la puissance 3, nous allons tomber sur la solution qui existe soit par addition soit par soustraction. p = √zN et q = √yN .
    Alors [ p² – q² = (xN) ] = [ zN – yN = (xN) ].

    Lorsque p est mis au carré, il devient un cube ainsi que q par conséquent p² + q² = z’ > z^N entier, donc racine carrée d’un cube au carré, z’ est un cube : z’^N, c’est aussi ce que l’on recherche par supposition, et nous savons que ce cas est impossible dans N = 2 et 4, deux solutions directes.

    (« Cela sous entendrait qu’il existe p’< p et q’ < q mais aussi p’’ < p’ < p et q’’ < q’ < q puisqu’il y aurait deux solutions inférieures ; donc une infinité de racine carrée de cube.. ? »)

    On sait aussi que (x^3)² = (z’^3 + y) (z’^3 – y) et (z’^3) – y = d² ce dernier est un entier,(« car (x^3)² ne serait pas un carré parfait ») y, est donc un entier par supposition et comme il est égal à 2pq, soit 2(√z^N) (√y^N) ; qui doit être aussi la racine carrée d’un cube, c’est ce que l’on recherche, y , est donc aussi un cube .

    On retrouve exactement la même contradiction que N = 2,4, 6..
    Et d² = (d^3) ² car bien évidemment (x^3)² n’aurait aucune chance de pouvoir exister.
    [« ou d’une autre façon : si (x^3)² = (z’^3 + y) (z’^3 – y), avec (z’^3 + y) et (z’^3 – y) premier entre eux, de parité différente, alors ces deux nombres sont aussi des cubes, et nécessairement, des cubes au carrés, puisque : (x^3)² = (x²)^3 est un carré !

    Par conséquent il existe p’<p et q’< q, qui nous donne la racine carée de la √z^N et aussi de la √y^N indéfiniment pythagorique, puisque z^N + y^N = z’^N ? et z^N – y^N = (x^N) . Nous voyons bien la contradiction générale de la démonstration n°2 et n° 1 du cas N = 4 ! Car z’^3 – y’^3 = (x^N) = (d²)^3 ne peut pas exister ; si Z est un produit de puissance première alors Y ne peut pas en être un !

    On en conclu donc que : (z’^3 + y) n’est pas un cube ni la racine carrée d’un cube, ainsi que : (z’^3 – y) quand bien même le produit de ces deux nombres serait égal à (x^3)² ou sa racine carrée ! Il en serait de même pour une puissance N > 3»]

    D’où on se retrouverait avec le T.P : (x^N), (y^N) et (z’^N) qui nous donne la solution (x^3)² + (y^3)² = (z’^3)² soit A^3 + B^3= C^3, ainsi que x^6 + y^6 = z^6 mais aussi U² + V² = E²
    ce qui a été démontré impossible avec p et q non entier, cela donnerait trois carrés A, B et C dans un Triplet non pythagorique, avec les facteurs qui seraient aussi trois carrés.
    et sans oublier le T.P p² + q² = (z’^3) qui nous donne z^3 + y^3 = z’^3 par supposition, avec le T.P primitif où p et q ont été choisis ; ce qui obligatoirement nous donnerait deux cubes = un cube par addition et soustraction , d’une part avec p et q et d’autre part AVEC: (z’^3 + y) (z’^3 – y) !

    Par conséquent on ne peut pas avoir choisi p et q, dans ce T.P primitif qui existerait ! même s-il n’en existait qu’un seul ;
    et la racine cubique de [2(√z^N) (√y^N]2 = y^N par supposition, n’est pas un entier ni la racine carrée d’un produit de puissance > 2! Alors il n’existe aucun T.P dans les racines carrées entières ou non des entiers élevés à la puissance première ≥ 3.

    Il en serait de même pour toute puissance première > 3 ; puisqu’il n’existe pas de T.P de racine carrée d’une puissance première >2, où p et q pourraient être choisis, et par obligation nous donner toutes ces solutions impossibles et contradictoires !

    Si 2(√z^N) (√y^N) = y’ n’est pas entier, il doit être une racine carrée d’un cube. Le T.P mis au carré On obtient : x^6 + y’^3 = z^6, = , A^3 + y’^3 = C^3 , = , U² + y’^3 = E² ; en plus et obligatoirement la solution impossible z^3 + y^3 = Z^3 tout aussi contradictoire que le cas N=2 et 4 et où la racine cubique de y’^3 ne peut en aucun cas être un entier, ni pour cet exemple une racine carrée d’un cube !

    (Mais pour N =2, on peut choisir p et q dans un T.P de racine carrée qui existe et avoir une solution > dans N = 2 uniquement ! ce qui serait impossible dans une puissance N > 2 et première, sans donner une solution dans le carré de cette puissance)

    Il n’y a donc aucune solution de manière générale quelque ne soit une puissance paire ou impair !
    Il est évident que p et q ne peut pas donner trois racines carrées non entières d’une puissance N ≥ 3 ; car si il existe une et une seule solution dans N > 2 le T.P ne peut être constitué qu’avec un seul entier à la puissance N. Comme pour N = 2 où là il ne peut y avoir qu’un carré mais trois entiers!

    Alors Fermat n’avait plus qu’à trouver, d’une autre façon, pourquoi p et q ne pouvaient donner trois racines carrées d’entiers à la puissance 3 ou 5 … peu importe. En démontrant le cas N = 3 de deux façons différentes, il démontrait qu’effectivement, quel que soit la nature de p et q, on ne peut pas constituer un T.P de trois racines carrées, ce qui confirme le même cas pour N = 2 et 4, si p et q n’est pas entier il a donc généralisé.

    Le T.P si il existe malgré tout , il est constitué de deux racines carrées non entières au minimum.
    Par conséquent il existe encore p’< p et q’<q qui a donné p et q pythagorique, soit : p = √z^N et q = √y^N, puisqu’il existerait p² – q² = xN ou l’inverse avec zN.
    Mais p’ et q’ ne sont donc plus pythagoriques ; ou alors ils le sont indéfiniment avec une infinité de solution dans N > 2 ce qui est absurde.

    Or p’ et q’, (comme on l’a fait pour N = 2) sont choisis dans le même niveau de racines carrées que p et q mais plus petits, ce qui est impossible !

    En effet p’ et q’ non entiers, seraient quand même deux racines carrées : √p’^N et √q’^N qui doivent donner p et q pythagorique ainsi que x, soit √z^N, √y^N et √x^N.
    Or p’ et q’ au carré sont deux cubes entiers, (ou pareille pour une autre puissance) qui redonnent deux solutions dans N, si √z^N et √x^N sont deux entiers et c'est justement ce qui va se produire; soit : p’² + q’² = √z^N et p’² – q’² = √x^N, c’est à dire : p’^N + q’^N = √z^N entier et p’^N – q’^N = √x^N entier
    « donc il ne pourrait y avoir dans une puissance N >2 une et uniquement une seule solution, au minimum deux qui en donnerait une infinité en continuant ! » Ainsi que 2 √p’^N √q’^N = √y^N non entier ! et bien sur :
    p’^N² + q’^N² ≠ z^N et p’^N² – q’^N² ≠ x^N, car cela existerait indéfiniment, et au minimum dans N = 3 et N = 2. Donc √p’^N et √q’^N ne peut pas donner p > p’ et q > q’, pythagorique pour N >2, ni √x^N ou √z^N ; et encore moins si p’ et q’ étaient choisis entre ces racines carrées de N >2 ; (puisque mis au carré, on ne pourrait obtenir le produit de puissance N recherché !). D’où l’absence de solution générale quel que soit N >2, par induction ce qui est vrai pour N = 2 et 4 est vrai pour N + 1 et 2..etc.

    Ceci termine mon raisonnement sur la solution générale de ce théorème.

    Il me paraît donc évident, que P de Fermat ne pouvait ignorer cette généralité, ne serait-ce que pour les puissances paires ! Ce qui n’a jamais été fait ! Il n’avait donc pas besoin de la propriété de la divisibilité dans N = 4 ni de sa méthode de descente infinie pour ce cas N = 4!

  23. #53
    invitea8961440

    Cool Re : triplet pythagoricien.

    Citation Envoyé par leg
    bonsoir à tous.
    suite poste 51.

    P de Fermat c’est arrêté au cas N = 3, prenons cet exemple, il est général. Si une solution existe dans N = 3 , p et q étant choisis dans les racines carrées de ces entiers à la puissance 3, nous allons tomber sur la solution qui existe soit par addition soit par soustraction. p = √zN et q = √yN .
    Alors [ p² – q² = (xN) ] = [ zN – yN = (xN) ].

    Lorsque p est mis au carré, il devient un cube ainsi que q par conséquent p² + q² = z’ > z^N entier, donc racine carrée d’un cube au carré, z’ est un cube : z’^N, c’est aussi ce que l’on recherche par supposition, et nous savons que ce cas est impossible dans N = 2 et 4, deux solutions directes.
    j'ai pris l'habitude de lire des démonstrations concises car me dis-je et si je lisais et que en cours de chemin c'était faux ,tout cela ne concordait pas!
    (« Cela sous entendrait qu’il existe p’< p et q’ < q mais aussi p’’ < p’ < p et q’’ < q’ < q puisqu’il y aurait deux solutions inférieures ; donc une infinité de racine carrée de cube.. ? »)

    On sait aussi que (x^3)² = (z’^3 + y) (z’^3 – y) et (z’^3) – y = d² ce dernier est un entier,(« car (x^3)² ne serait pas un carré parfait ») y, est donc un entier par supposition et comme il est égal à 2pq, soit 2(√z^N) (√y^N) ; qui doit être aussi la racine carrée d’un cube, c’est ce que l’on recherche, y , est donc aussi un cube .

    On retrouve exactement la même contradiction que N = 2,4, 6..
    Et d² = (d^3) ² car bien évidemment (x^3)² n’aurait aucune chance de pouvoir exister.
    [« ou d’une autre façon : si (x^3)² = (z’^3 + y) (z’^3 – y), avec (z’^3 + y) et (z’^3 – y) premier entre eux, de parité différente, alors ces deux nombres sont aussi des cubes, et nécessairement, des cubes au carrés, puisque : (x^3)² = (x²)^3 est un carré !

    Par conséquent il existe p’<p et q’< q, qui nous donne la racine carée de la √z^N et aussi de la √y^N indéfiniment pythagorique, puisque z^N + y^N = z’^N ? et z^N – y^N = (x^N) . Nous voyons bien la contradiction générale de la démonstration n°2 et n° 1 du cas N = 4 ! Car z’^3 – y’^3 = (x^N) = (d²)^3 ne peut pas exister ; si Z est un produit de puissance première alors Y ne peut pas en être un !

    On en conclu donc que : (z’^3 + y) n’est pas un cube ni la racine carrée d’un cube, ainsi que : (z’^3 – y) quand bien même le produit de ces deux nombres serait égal à (x^3)² ou sa racine carrée ! Il en serait de même pour une puissance N > 3»]

    D’où on se retrouverait avec le T.P : (x^N), (y^N) et (z’^N) qui nous donne la solution (x^3)² + (y^3)² = (z’^3)² soit A^3 + B^3= C^3, ainsi que x^6 + y^6 = z^6 mais aussi U² + V² = E²
    ce qui a été démontré impossible avec p et q non entier, cela donnerait trois carrés A, B et C dans un Triplet non pythagorique, avec les facteurs qui seraient aussi trois carrés.
    et sans oublier le T.P p² + q² = (z’^3) qui nous donne z^3 + y^3 = z’^3 par supposition, avec le T.P primitif où p et q ont été choisis ; ce qui obligatoirement nous donnerait deux cubes = un cube par addition et soustraction , d’une part avec p et q et d’autre part AVEC: (z’^3 + y) (z’^3 – y) !

    Par conséquent on ne peut pas avoir choisi p et q, dans ce T.P primitif qui existerait ! même s-il n’en existait qu’un seul ;
    et la racine cubique de [2(√z^N) (√y^N]2 = y^N par supposition, n’est pas un entier ni la racine carrée d’un produit de puissance > 2! Alors il n’existe aucun T.P dans les racines carrées entières ou non des entiers élevés à la puissance première ≥ 3.

    Il en serait de même pour toute puissance première > 3 ; puisqu’il n’existe pas de T.P de racine carrée d’une puissance première >2, où p et q pourraient être choisis, et par obligation nous donner toutes ces solutions impossibles et contradictoires !

    Si 2(√z^N) (√y^N) = y’ n’est pas entier, il doit être une racine carrée d’un cube. Le T.P mis au carré On obtient : x^6 + y’^3 = z^6, = , A^3 + y’^3 = C^3 , = , U² + y’^3 = E² ; en plus et obligatoirement la solution impossible z^3 + y^3 = Z^3 tout aussi contradictoire que le cas N=2 et 4 et où la racine cubique de y’^3 ne peut en aucun cas être un entier, ni pour cet exemple une racine carrée d’un cube !

    (Mais pour N =2, on peut choisir p et q dans un T.P de racine carrée qui existe et avoir une solution > dans N = 2 uniquement ! ce qui serait impossible dans une puissance N > 2 et première, sans donner une solution dans le carré de cette puissance)

    Il n’y a donc aucune solution de manière générale quelque ne soit une puissance paire ou impair !
    Il est évident que p et q ne peut pas donner trois racines carrées non entières d’une puissance N ≥ 3 ; car si il existe une et une seule solution dans N > 2 le T.P ne peut être constitué qu’avec un seul entier à la puissance N. Comme pour N = 2 où là il ne peut y avoir qu’un carré mais trois entiers!

    Alors Fermat n’avait plus qu’à trouver, d’une autre façon, pourquoi p et q ne pouvaient donner trois racines carrées d’entiers à la puissance 3 ou 5 … peu importe. En démontrant le cas N = 3 de deux façons différentes, il démontrait qu’effectivement, quel que soit la nature de p et q, on ne peut pas constituer un T.P de trois racines carrées, ce qui confirme le même cas pour N = 2 et 4, si p et q n’est pas entier il a donc généralisé.

    Le T.P si il existe malgré tout , il est constitué de deux racines carrées non entières au minimum.
    Par conséquent il existe encore p’< p et q’<q qui a donné p et q pythagorique, soit : p = √z^N et q = √y^N, puisqu’il existerait p² – q² = xN ou l’inverse avec zN.
    Mais p’ et q’ ne sont donc plus pythagoriques ; ou alors ils le sont indéfiniment avec une infinité de solution dans N > 2 ce qui est absurde.

    Or p’ et q’, (comme on l’a fait pour N = 2) sont choisis dans le même niveau de racines carrées que p et q mais plus petits, ce qui est impossible !

    En effet p’ et q’ non entiers, seraient quand même deux racines carrées : √p’^N et √q’^N qui doivent donner p et q pythagorique ainsi que x, soit √z^N, √y^N et √x^N.
    Or p’ et q’ au carré sont deux cubes entiers, (ou pareille pour une autre puissance) qui redonnent deux solutions dans N, si √z^N et √x^N sont deux entiers et c'est justement ce qui va se produire; soit : p’² + q’² = √z^N et p’² – q’² = √x^N, c’est à dire : p’^N + q’^N = √z^N entier et p’^N – q’^N = √x^N entier
    « donc il ne pourrait y avoir dans une puissance N >2 une et uniquement une seule solution, au minimum deux qui en donnerait une infinité en continuant ! » Ainsi que 2 √p’^N √q’^N = √y^N non entier ! et bien sur :
    p’^N² + q’^N² ≠ z^N et p’^N² – q’^N² ≠ x^N, car cela existerait indéfiniment, et au minimum dans N = 3 et N = 2. Donc √p’^N et √q’^N ne peut pas donner p > p’ et q > q’, pythagorique pour N >2, ni √x^N ou √z^N ; et encore moins si p’ et q’ étaient choisis entre ces racines carrées de N >2 ; (puisque mis au carré, on ne pourrait obtenir le produit de puissance N recherché !). D’où l’absence de solution générale quel que soit N >2, par induction ce qui est vrai pour N = 2 et 4 est vrai pour N + 1 et 2..etc.

    Ceci termine mon raisonnement sur la solution générale de ce théorème.

    Il me paraît donc évident, que P de Fermat ne pouvait ignorer cette généralité, ne serait-ce que pour les puissances paires ! Ce qui n’a jamais été fait ! Il n’avait donc pas besoin de la propriété de la divisibilité dans N = 4 ni de sa méthode de descente infinie pour ce cas N = 4!
    j'ai pris l'habitude de lire des démonstarations concises ,si tu as pu retrouver la démonstration de fermat de son dernier theoreme ,je presume en 3 pages,pourquoi ne pas le publier!

  24. #54
    invitea8961440

    Cool Re : triplet pythagoricien.

    Citation Envoyé par leg
    bonsoir à tous.
    suite poste 51.

    P de Fermat c’est arrêté au cas N = 3, prenons cet exemple, il est général. Si une solution existe dans N = 3 , p et q étant choisis dans les racines carrées de ces entiers à la puissance 3, nous allons tomber sur la solution qui existe soit par addition soit par soustraction. p = √zN et q = √yN .
    Alors [ p² – q² = (xN) ] = [ zN – yN = (xN) ].

    Lorsque p est mis au carré, il devient un cube ainsi que q par conséquent p² + q² = z’ > z^N entier, donc racine carrée d’un cube au carré, z’ est un cube : z’^N, c’est aussi ce que l’on recherche par supposition, et nous savons que ce cas est impossible dans N = 2 et 4, deux solutions directes.
    j'ai pris l'habitude de lire des démonstrations concises car me dis-je et si je lisais et que en cours de chemin c'était faux ,tout cela ne concordait pas!
    (« Cela sous entendrait qu’il existe p’< p et q’ < q mais aussi p’’ < p’ < p et q’’ < q’ < q puisqu’il y aurait deux solutions inférieures ; donc une infinité de racine carrée de cube.. ? »)

    On sait aussi que (x^3)² = (z’^3 + y) (z’^3 – y) et (z’^3) – y = d² ce dernier est un entier,(« car (x^3)² ne serait pas un carré parfait ») y, est donc un entier par supposition et comme il est égal à 2pq, soit 2(√z^N) (√y^N) ; qui doit être aussi la racine carrée d’un cube, c’est ce que l’on recherche, y , est donc aussi un cube .

    On retrouve exactement la même contradiction que N = 2,4, 6..
    Et d² = (d^3) ² car bien évidemment (x^3)² n’aurait aucune chance de pouvoir exister.
    [« ou d’une autre façon : si (x^3)² = (z’^3 + y) (z’^3 – y), avec (z’^3 + y) et (z’^3 – y) premier entre eux, de parité différente, alors ces deux nombres sont aussi des cubes, et nécessairement, des cubes au carrés, puisque : (x^3)² = (x²)^3 est un carré !

    Par conséquent il existe p’<p et q’< q, qui nous donne la racine carée de la √z^N et aussi de la √y^N indéfiniment pythagorique, puisque z^N + y^N = z’^N ? et z^N – y^N = (x^N) . Nous voyons bien la contradiction générale de la démonstration n°2 et n° 1 du cas N = 4 ! Car z’^3 – y’^3 = (x^N) = (d²)^3 ne peut pas exister ; si Z est un produit de puissance première alors Y ne peut pas en être un !

    On en conclu donc que : (z’^3 + y) n’est pas un cube ni la racine carrée d’un cube, ainsi que : (z’^3 – y) quand bien même le produit de ces deux nombres serait égal à (x^3)² ou sa racine carrée ! Il en serait de même pour une puissance N > 3»]

    D’où on se retrouverait avec le T.P : (x^N), (y^N) et (z’^N) qui nous donne la solution (x^3)² + (y^3)² = (z’^3)² soit A^3 + B^3= C^3, ainsi que x^6 + y^6 = z^6 mais aussi U² + V² = E²
    ce qui a été démontré impossible avec p et q non entier, cela donnerait trois carrés A, B et C dans un Triplet non pythagorique, avec les facteurs qui seraient aussi trois carrés.
    et sans oublier le T.P p² + q² = (z’^3) qui nous donne z^3 + y^3 = z’^3 par supposition, avec le T.P primitif où p et q ont été choisis ; ce qui obligatoirement nous donnerait deux cubes = un cube par addition et soustraction , d’une part avec p et q et d’autre part AVEC: (z’^3 + y) (z’^3 – y) !

    Par conséquent on ne peut pas avoir choisi p et q, dans ce T.P primitif qui existerait ! même s-il n’en existait qu’un seul ;
    et la racine cubique de [2(√z^N) (√y^N]2 = y^N par supposition, n’est pas un entier ni la racine carrée d’un produit de puissance > 2! Alors il n’existe aucun T.P dans les racines carrées entières ou non des entiers élevés à la puissance première ≥ 3.

    Il en serait de même pour toute puissance première > 3 ; puisqu’il n’existe pas de T.P de racine carrée d’une puissance première >2, où p et q pourraient être choisis, et par obligation nous donner toutes ces solutions impossibles et contradictoires !

    Si 2(√z^N) (√y^N) = y’ n’est pas entier, il doit être une racine carrée d’un cube. Le T.P mis au carré On obtient : x^6 + y’^3 = z^6, = , A^3 + y’^3 = C^3 , = , U² + y’^3 = E² ; en plus et obligatoirement la solution impossible z^3 + y^3 = Z^3 tout aussi contradictoire que le cas N=2 et 4 et où la racine cubique de y’^3 ne peut en aucun cas être un entier, ni pour cet exemple une racine carrée d’un cube !

    (Mais pour N =2, on peut choisir p et q dans un T.P de racine carrée qui existe et avoir une solution > dans N = 2 uniquement ! ce qui serait impossible dans une puissance N > 2 et première, sans donner une solution dans le carré de cette puissance)

    Il n’y a donc aucune solution de manière générale quelque ne soit une puissance paire ou impair !
    Il est évident que p et q ne peut pas donner trois racines carrées non entières d’une puissance N ≥ 3 ; car si il existe une et une seule solution dans N > 2 le T.P ne peut être constitué qu’avec un seul entier à la puissance N. Comme pour N = 2 où là il ne peut y avoir qu’un carré mais trois entiers!

    Alors Fermat n’avait plus qu’à trouver, d’une autre façon, pourquoi p et q ne pouvaient donner trois racines carrées d’entiers à la puissance 3 ou 5 … peu importe. En démontrant le cas N = 3 de deux façons différentes, il démontrait qu’effectivement, quel que soit la nature de p et q, on ne peut pas constituer un T.P de trois racines carrées, ce qui confirme le même cas pour N = 2 et 4, si p et q n’est pas entier il a donc généralisé.

    Le T.P si il existe malgré tout , il est constitué de deux racines carrées non entières au minimum.
    Par conséquent il existe encore p’< p et q’<q qui a donné p et q pythagorique, soit : p = √z^N et q = √y^N, puisqu’il existerait p² – q² = xN ou l’inverse avec zN.
    Mais p’ et q’ ne sont donc plus pythagoriques ; ou alors ils le sont indéfiniment avec une infinité de solution dans N > 2 ce qui est absurde.

    Or p’ et q’, (comme on l’a fait pour N = 2) sont choisis dans le même niveau de racines carrées que p et q mais plus petits, ce qui est impossible !

    En effet p’ et q’ non entiers, seraient quand même deux racines carrées : √p’^N et √q’^N qui doivent donner p et q pythagorique ainsi que x, soit √z^N, √y^N et √x^N.
    Or p’ et q’ au carré sont deux cubes entiers, (ou pareille pour une autre puissance) qui redonnent deux solutions dans N, si √z^N et √x^N sont deux entiers et c'est justement ce qui va se produire; soit : p’² + q’² = √z^N et p’² – q’² = √x^N, c’est à dire : p’^N + q’^N = √z^N entier et p’^N – q’^N = √x^N entier
    « donc il ne pourrait y avoir dans une puissance N >2 une et uniquement une seule solution, au minimum deux qui en donnerait une infinité en continuant ! » Ainsi que 2 √p’^N √q’^N = √y^N non entier ! et bien sur :
    p’^N² + q’^N² ≠ z^N et p’^N² – q’^N² ≠ x^N, car cela existerait indéfiniment, et au minimum dans N = 3 et N = 2. Donc √p’^N et √q’^N ne peut pas donner p > p’ et q > q’, pythagorique pour N >2, ni √x^N ou √z^N ; et encore moins si p’ et q’ étaient choisis entre ces racines carrées de N >2 ; (puisque mis au carré, on ne pourrait obtenir le produit de puissance N recherché !). D’où l’absence de solution générale quel que soit N >2, par induction ce qui est vrai pour N = 2 et 4 est vrai pour N + 1 et 2..etc.

    Ceci termine mon raisonnement sur la solution générale de ce théorème.

    Il me paraît donc évident, que P de Fermat ne pouvait ignorer cette généralité, ne serait-ce que pour les puissances paires ! Ce qui n’a jamais été fait ! Il n’avait donc pas besoin de la propriété de la divisibilité dans N = 4 ni de sa méthode de descente infinie pour ce cas N = 4!
    J'ai pris l'habitude de lire des démonstarations concises ,si tu as pu retrouver la démonstration de fermat de son dernier theoreme ,je presume en 3 pages,pourquoi ne pas le publier!

  25. #55
    leg

    Re : triplet pythagoricien.

    bonsoir ulrich richarovitch.
    pour la publier il faudrait être vraiment mathématicien ce qui n'est pas mon cas.
    par contre je pense que ce forum de futura permettra peut être de le faire, et du moins d'avoir des avis ou d'étudier le raisonnement que j'ai utliser; mais mon but et avant tout de reposer l'histoire de Fermat et si cette démonstration avec son raisonnement et admise alors il sera impossible aux professionnels de dire que fermat n'avait pas la solution..la démonstration de ce théorème n'apporte rient de plus en mathématique, si ce n'est que l'histoire de Fermat . Maintenant il reste à trouver un contradiction a mon raisonnement ou à l'admettre... car si il y a une érreure, elle ne peut être qu'au niveau des puissances premières , 2.3.5....ect, personne n'a résolu les puissances paires de manière générale et élémentaire.
    il y a suffisament de matheux sur ce site pour avoir une idée précise et des commentaires constructifs, je vous laisse la parole en attendant et merci pour vos remarques...
    un dernier petit mot, pour une démonstration concise il s'agit avant tout d'un raisonnement qui ne se met pas en équation; tout comme le raisonnement de fermat: si on donne deux carrés on obtiendra deux carrés plus petits ayant la même propriété ...etc etc sur qu'elle proriété il s'est appuyé ?. car on a pas ses écrits même pour le cas N=4, et personne n'a cherché à démontrer ce cas, des trois façons possible! les contradictions des puissances paires sont triviales et n'utilisent qu'une propriété, le reste n'en est que la suite logique. A++ leg.

  26. #56
    leg

    Re : triplet pythagoricien.

    bonjour et suite de ma conclusion.

    En résumé :


    Son idée merveilleuse, est en définitive très simple :
    où choisir p et q, afin d’obtenir un T.P dans une puissance première, ou encore de reconstituer un T.P supposé existé !
    Tout naturellement dans les racines carrées de ces produits de puissance N, premier. Ce qui est possible pour les T.P ayant une solution dans N = 2 ;
    mais cela devient impossible pour N =3, 5, 7, 11…sans soulever de contradiction, tel que : deux cubes donneraient un cube par addition et soustraction ou plus généralement deux produits de puissance N premier, donneraient deux autres produits ayant la même propriété, par addition et soustraction, ce qui est impossible ! Mais un T.P dans les racines carrées de N = 3, 5… en implique une infinité, « mais non entiers » non pas en utilisant un facteur commun, mais tout simplement en construisant un losange, en partant de l’hypoténuse du triangle rectangle, dont la surface est égal à la somme des surfaces des deux triangles rectangles, construits sur les deux autres côtés.

    Le raisonnement de P de Fermat était si on donne deux entiers à la puissance deux, sous entendu:, 3 ,4 ,5 ..etc, on obtient par addition et soustraction la même propriété, ce qui est impossible par définition ! Or si et seulement si il y avait eu, une solution dans une de ces puissances premières > 2 ; le Triplet Pythagoricien se trouvait obligatoirement dans les racines carrées, comme pour la puissance 2, et en choisissant p et q dans les racines carrées de ces puissances Premières > 2 afin de constituer le T.P supposé existé, on obtient la contradiction recherchée !
    P² - q² = (x^N) et P² + q² = (z^N) qui est égal a : p^N – q^N = (xN) et : pN + qN = (zN). Par conséquent il n’existe pas de T.P dans ces racines carrées !
    Deux solutions directes comme pour le cas N = 4, d’où il est impossible de choisir p et q dans ces racines carrées, afin de reconstituer le T.P supposé existé et donnant une solution dans N = 3 , 4 , 5 ,6 , 7……n premier ou pas car il existerait indéfiniment p’ < p et q’ < q donnant x’^N < (x^N) et z’^N < (z^N) !

  27. #57
    leg

    Re : triplet pythagoricien.

    bonne année a tous.
    Mais est ce que Fermat savait, qu' avec les Triplets Pythagoriciens,
    on pouvait touver la formule donnant l'infinité des scalènes ? :
    " triangles dont les trois côtés inégaux ainsi que l'aire se mesurent en entier"
    exemple le triplet Héronien d'un trianble a, b , c : 13 , 14 et 15 ; aire = 84.
    réponse : Non ...!
    d'ailleurs aucun mathématicien connu non plus !

    trouver un triplet H, d'un triangle quelconque a , b ,c dont l'aire est = au périmètre .
    A bientôt..

Page 2 sur 2 PremièrePremière 2

Discussions similaires

  1. noeud pythagoricien
    Par invitee21bc376 dans le forum Mathématiques du supérieur
    Réponses: 0
    Dernier message: 18/06/2007, 16h58
  2. le triplet statistique
    Par invite277087f0 dans le forum Mathématiques du supérieur
    Réponses: 2
    Dernier message: 04/05/2007, 10h25
  3. triplet de focale très courte / retroprojo3D
    Par invite4aeae10e dans le forum Matériel astronomique et photos d'amateurs
    Réponses: 0
    Dernier message: 10/12/2005, 22h30